Integreren bij sferische coordinaten

Moderators: dirkwb, Xilvo

Reageer
Berichten: 45

Integreren bij sferische coordinaten

Als we bevoorbeeld het volume onder z= x^2+y^2 (dus tussen het XY vlak en dit oppervlak) willen vinden in sferische coordinaten, hoe moeten we dit dan doen? (ik weet dat het eenvoudig is in cilindrische/cartesiaanse, maar ik zou ook graag weten hoe je het in sferische kan doen)

0<phi<2*Pi

Maar ik raak er niet aan uit hoe ik de grenzen van rho en theta kan vinden....

Alvast bedankt!

Berichten: 11

Re: Integreren bij sferische coordinaten

Het lijkt me dat
\( 0\leq \theta <2\pi \)
, met de notatie van
\(\theta \)
,
\( \phi \)
en
\(r\)
van op http://nl.wikipedia.org/wiki/Bolco%C3%B6rdinaten.

Voor vaste
\(\phi \)
en
\(r\)
hebben we nu een cirkel beschreven die in de halve sfeer ligt. Tussen welke waarden moeten
\(\phi \)
en
\(r\)
variëren om het gevraagde volume te beschrijven?

PS: ik ben ook een burgie in Gent. Welke richting doe je?

Berichten: 7.068

Re: Integreren bij sferische coordinaten

Als we bevoorbeeld het volume onder z= x^2+y^2 (dus tussen het XY vlak en dit oppervlak) willen vinden in sferische coordinaten, hoe moeten we dit dan doen?
Ik denk dat je zonder te rekenen al kan inzien dat dat volume niet begrensd is in grootte.

Wat ik zou doen:
\(x = r \cdot \sin(\theta) \cdot \cos(\phi)\)
\(y = r \cdot \sin(\theta) \cdot \sin(\phi)\)
\(z = r \cdot \cos(\theta)\)
\(0 \leq z \leq x^2 + y^2\)
\(0 \leq r \cdot \cos(\theta) \leq (r \cdot \sin(\theta) \cdot \cos(\phi))^2 + ( r \cdot \sin(\theta) \cdot \sin(\phi))^2\)
\(0 \leq r \cdot \cos(\theta) \leq r^2 \cdot \sin^2(\theta)\)
\(0 \leq \cos(\theta) \leq r \cdot \sin^2(\theta)\)
\(0 \leq \frac{\cos(\theta)}{\sin^2(\theta)} \leq r\)
Ik denk dat het in dit geval het handig is om hier de ondergrens voor r uit te halen. De bovengrens voor r bestaat niet. Je hebt nu dus de grenzen voor phi, voor r en voor theta zou ook wel moeten lukken.

Berichten: 45

Re: Integreren bij sferische coordinaten

Oops, vergeten te vermelden dat het volme ook nog binnen de cilinder met straal 1 rond de z as moet liggen (zodat het wel degelijk begrensd wordt).

dus de extra voorwaarde wordt:

x²+y²<1

-> r< 1/sin(theta)

of

z<1

-> r<1/cos(theta)

Welke is nu de juiste bovengrens? ;)

Ook lukt het nog steeds niet om een bovengrens voor theta (ondergrens=0) te vinden die onafhankelijk is van r.

PS.: ik heb hier theta gebruikt zoals EvilBro, dus niet zoals wikipedia

@tuure: bouwkunde

Reageer